Telescoping series convergence question

Click For Summary
The discussion centers on the convergence of the series Σ(n=1 to ∞) 8/(n(n+2)). The user attempts to decompose the series into Σ(n=1 to ∞)(8/n - 8/(n+2)) and calculates the sum, arriving at 12. However, they express confusion as the answer differs from the textbook solution. A response highlights a potential error in the user's algebra, indicating a miscalculation in the decomposition. The importance of proper parentheses in mathematical expressions is also emphasized.
SYoungblood
Messages
64
Reaction score
1

Homework Statement


[/B]
Hello, this problem is from a well-known calc text:

Σ(n=1 to ∞) 8/(n(n+2)

Homework Equations


[/B]
What I have here is decomposingg the problem into Σ(n=1 to ∞)(8/n -(8/n+2)

The Attempt at a Solution



I have the series sum as equaling (8/1-8/3) + (8/2-8/4) + (8/3-8/5) + (8/4-8/6) + … + (8/n-(8/n+2)

So, I have everything canceling but 8/1, 8/2, and 8/n+2. The last arm reaches 0 as n approaches ∞, so I have 8/1 + 8/2 = 12.

However, I think I am missing something here, the answer in the back of the book is not 12.

Thank you all for your help in advance.

SY
 
Physics news on Phys.org
SYoungblood said:

Homework Statement


[/B]
Hello, this problem is from a well-known calc text:

Σ(n=1 to ∞) 8/(n(n+2))

Homework Equations


[/B]
What I have here is decomposing the problem into Σ(n=1 to ∞)(8/n -(8/(n+2))

The Attempt at a Solution



I have the series sum as equaling (8/1-8/3) + (8/2-8/4) + (8/3-8/5) + (8/4-8/6) + … + (8/n-(8/(n+2))

So, I have everything canceling but 8/1, 8/2, and 8/(n+2). The last arm reaches 0 as n approaches ∞, so I have 8/1 + 8/2 = 12.

However, I think I am missing something here, the answer in the back of the book is not 12.

Thank you all for your help in advance.

SY
Check your algebra.

##\displaystyle \ \frac{8}{n}-\frac{8}{n+2}=\frac{8n+16-8n}{n(n+2)} \ ##

##\displaystyle \ \quad \quad \ne\frac{8}{n(n+2)} \ ##​

Also, be sure to use an adequate amount of parentheses.
 
  • Like
Likes SYoungblood
Question: A clock's minute hand has length 4 and its hour hand has length 3. What is the distance between the tips at the moment when it is increasing most rapidly?(Putnam Exam Question) Answer: Making assumption that both the hands moves at constant angular velocities, the answer is ## \sqrt{7} .## But don't you think this assumption is somewhat doubtful and wrong?

Similar threads

  • · Replies 3 ·
Replies
3
Views
1K
  • · Replies 3 ·
Replies
3
Views
2K
  • · Replies 7 ·
Replies
7
Views
2K
Replies
2
Views
1K
  • · Replies 1 ·
Replies
1
Views
2K
  • · Replies 1 ·
Replies
1
Views
2K
  • · Replies 4 ·
Replies
4
Views
2K
  • · Replies 4 ·
Replies
4
Views
1K
  • · Replies 2 ·
Replies
2
Views
4K
  • · Replies 5 ·
Replies
5
Views
2K